Gull and Gill's theory

Foundations of physics and/or philosophy of physics, and in particular, posts on unresolved or controversial issues

Re: Gull and Gill's theory

Postby Justo » Mon May 10, 2021 5:10 am

FrediFizzx wrote:It is really hard to believe that any mathematician would believe such nonsense. The A1 in the first pair is impossible to be the same as the A1 in the second pair, etc. So, you could have,

A1 B1= +1, A1 B2= -1, A2 B1= -1, A2 B2= -1 --> +1+1+1+1 = 4

Bell was wrong, get over it and move on!!!!!
.

I agree with @FredFizzx, there is an interesting article by Adenier that I like because it explains this problem very clearly https://arxiv.org/abs/quant-ph/0006014. It was published in a Conference Proceedings Foundations of Probability and Physics, chapter Refutation of Bell’s Theorem, pages 29–38. World Scientific, 2001.

However, when the Bell derivation is correctly interpreted that problem does not exist.
Justo
 

Re: Gull and Gill's theory

Postby FrediFizzx » Mon May 10, 2021 6:05 am

Justo wrote:
FrediFizzx wrote:It is really hard to believe that any mathematician would believe such nonsense. The A1 in the first pair is impossible to be the same as the A1 in the second pair, etc. So, you could have,

A1 B1= +1, A1 B2= -1, A2 B1= -1, A2 B2= -1 --> +1+1+1+1 = 4

Bell was wrong, get over it and move on!!!!!
.

I agree with @FredFizzx, there is an interesting article by Adenier that I like because it explains this problem very clearly https://arxiv.org/abs/quant-ph/0006014. It was published in a Conference Proceedings Foundations of Probability and Physics, chapter Refutation of Bell’s Theorem, pages 29–38. World Scientific, 2001.

However, when the Bell derivation is correctly interpreted that problem does not exist.

Apparently you don't believe or understand our proof that shoots down Bell's theory (interpretation). If any questions or comments about our proof, please ask. I would be happy to explain it to you,

Image
Image
.
FrediFizzx
Independent Physics Researcher
 
Posts: 2905
Joined: Tue Mar 19, 2013 7:12 pm
Location: N. California, USA

Re: Gull and Gill's theory

Postby Joy Christian » Mon May 10, 2021 6:06 am

Justo wrote:
However, when the Bell derivation is correctly interpreted that problem does not exist.

Anything and everything can be "correctly interpreted" to justify the conclusion we already believe in. That is why Bell's theorem is a belief system, not science.
.
Joy Christian
Research Physicist
 
Posts: 2793
Joined: Wed Feb 05, 2014 4:49 am
Location: Oxford, United Kingdom

Re: Gull and Gill's theory

Postby Justo » Mon May 10, 2021 6:49 am

FrediFizzx wrote: Apparently you don't believe or understand our proof that shoots down Bell's theory (interpretation). If any questions or comments about our proof, please ask. I would be happy to explain it to you,
.

That's right. I can't understand because can't read your code. I am not good at computer programs. But I am curious. Please give some reference to your computer language and maybe will try to learn it whenever I find the time and then get back to you asking questions.
Justo
 

Re: Gull and Gill's theory

Postby FrediFizzx » Mon May 10, 2021 6:54 am

Justo wrote:
FrediFizzx wrote: Apparently you don't believe or understand our proof that shoots down Bell's theory (interpretation). If any questions or comments about our proof, please ask. I would be happy to explain it to you,
.

That's right. I can't understand because can't read your code. I am not good at computer programs. But I am curious. Please give some reference to your computer language and maybe will try to learn it whenever I find the time and then get back to you asking questions.

https://www.wolfram.com/mathematica/

A lot of it is pretty much self-explanatory. The rest I can easily explain to you. What is the first part you don't understand?
.
FrediFizzx
Independent Physics Researcher
 
Posts: 2905
Joined: Tue Mar 19, 2013 7:12 pm
Location: N. California, USA

Re: Gull and Gill's theory

Postby Justo » Mon May 10, 2021 4:38 pm

FrediFizzx wrote:A lot of it is pretty much self-explanatory. The rest I can easily explain to you. What is the first part you don't understand?
.

Thank you. Another question, is there some kind of free version that one can use?
Justo
 

Re: Gull and Gill's theory

Postby FrediFizzx » Mon May 10, 2021 5:44 pm

Justo wrote:
FrediFizzx wrote:A lot of it is pretty much self-explanatory. The rest I can easily explain to you. What is the first part you don't understand?
.

Thank you. Another question, is there some kind of free version that one can use?

You can do a 15 day free trial.

https://www.wolfram.com/mathematica/trial/
.
FrediFizzx
Independent Physics Researcher
 
Posts: 2905
Joined: Tue Mar 19, 2013 7:12 pm
Location: N. California, USA

Re: Gull and Gill's theory

Postby FrediFizzx » Thu May 13, 2021 5:05 pm

It doesn't look like anything was fixed yet on the paper,

https://arxiv.org/abs/2012.00719

I already showed how to fix eq.(1) by specifying actual A and B measurement functions. Do you want me to show how to fix eq. (4)??? It is pretty easy. :mrgreen:
.
FrediFizzx
Independent Physics Researcher
 
Posts: 2905
Joined: Tue Mar 19, 2013 7:12 pm
Location: N. California, USA

Re: Gull and Gill's theory

Postby Joy Christian » Sun May 16, 2021 3:29 am

FrediFizzx wrote:It doesn't look like anything was fixed yet on the paper,

https://arxiv.org/abs/2012.00719

I already showed how to fix eq.(1) by specifying actual A and B measurement functions. Do you want me to show how to fix eq. (4)??? It is pretty easy. :mrgreen:

Gill is not coming back. Perhaps karma has finally caught up with him? In the irrational world in which a non-theorem like Bell's "theorem" holds, the law of karma would hold too!
.
Joy Christian
Research Physicist
 
Posts: 2793
Joined: Wed Feb 05, 2014 4:49 am
Location: Oxford, United Kingdom

Re: Gull and Gill's theory

Postby FrediFizzx » Sun May 16, 2021 4:52 am

Joy Christian wrote:
FrediFizzx wrote:It doesn't look like anything was fixed yet on the paper,

https://arxiv.org/abs/2012.00719

I already showed how to fix eq.(1) by specifying actual A and B measurement functions. Do you want me to show how to fix eq. (4)??? It is pretty easy. :mrgreen:

Gill is not coming back. Perhaps karma has finally caught up with him? In the irrational world in which a non-theorem like Bell's "theorem" holds, the law of karma would hold too!
.

Actually, eq. (1) still has a problem. It's non-local.
.
FrediFizzx
Independent Physics Researcher
 
Posts: 2905
Joined: Tue Mar 19, 2013 7:12 pm
Location: N. California, USA

Previous

Return to Sci.Physics.Foundations

Who is online

Users browsing this forum: ahrefs [Bot] and 76 guests

cron
CodeCogs - An Open Source Scientific Library